Preuve d'inégalité pour calcul de limite

Bonjour, il y a deux inégalités que je ne comprends pas. Peut-être ce sont des D.L. (les coeffs laissent cela supposer) mais je ne suis pas encore suffisamment habile pour réussir à les calculer. Avez-vous une idée ?
\begin{align*}
\frac{ ( |x|+|y|)^2 } { \sqrt {x^2 + y^2} } &\leq 4 \sqrt{x^2 + y^2} \\
\frac{ e^{ -\tfrac{1}{\sqrt {x^2 + y^2}} } }{ x^2 + y^2 } &< 6 \sqrt {x^2 + y^2}
\end{align*}
Pourriez-vous m'aider svp ? Et si ce sont des DL comment les calculer ?
Merci.

Réponses

  • Pour la première, multiplie des 2 côtés par $\sqrt{x^2+y^2}$ et utilise le fait que
    $$\forall a,b\in\mathbb R,\; 2ab\le a^2+b^2$$
    (ce qui donne une meilleure majoration d'ailleurs).
  • Pour la 1 : $(|x| + |y|)^{2} = x^{2} + 2|x||y| + y^{2} \leq 2x^{2} + 2y^{2}$. Donc $\frac{ ( |x|+|y|)^2 } { x^2 + y^2 } \leq 2 $ Donc en mutlipliant par $\sqrt{x^{2} + y^{2}}$ tu peux remplacer ton 4 par un 2. L'ingalité est à fortiori vrai.

    Pour la 2) que peux-tu dire quand $x, y$ tendent vers $0$? Cela semble coincer ici.
    À moins que tu sois loin de 0.
  • Et paf, grillé par paf.
  • Merci pour la 1! mais je suis étonné que le livre n'écrive pas 2 alors... Avez vous une idée de comment ils sont arrivés au résultat?


    Pour la deuxième, c'est justement mon objectif que de calculer la limite ensuite... mais On voit que c'est du x exp(-x) donc en +inf ça devrait donner 0. Mais je cherche une majoration pour le démontrer
  • Pour trouver ton 4 du livre de la 1 , utilise $|x|\leq \sqrt{x^2+y^2}$ et aussi $|y|\leq \sqrt{x^2+y^2}$
    Le 😄 Farceur


  • ah oui okay je comprends merci pour votre aide :).

    Si je peux encore avoir avoir les explications pour la deuxième ça serait génial
  • Pour la deuxieme tu poses $t=1/\sqrt{x^2+y^2}$ et tu as a demontrer que $f(t)=e^{-t}t^3<6$ si $t>0.$ Tu etudies le max de $f$ et tu verras qu'il est largement $<6.$
  • d'accord merci pour tout!
  • Ah oui ma limite n'est pas vraie. On oublie ;-).
  • Pour la 2, si la question était de calculer la limite en (0,0) de $$(x,y)\to \frac{e^{ -\tfrac{1}{\sqrt {x^2 + y^2}} } } { x^2 + y^2 }$$On peut la calculer sans passer par l’inégalité suggérée, il suffit de connaitre que la limite en $+\infty$ de $X\to X^2e^{-X} $ égale à 0
    Le 😄 Farceur


Connectez-vous ou Inscrivez-vous pour répondre.